• Anúncio Global
    Respostas
    Exibições
    Última mensagem

Questão, número irracional.

Questão, número irracional.

Mensagempor LuizCarlos » Sex Mar 16, 2012 18:49

Olá amigos!

Seguinte, estou fazendo exercícios, não estou entendendo essa questão:

Escreva a representação decimal de um número irracional compreendido entre 5 e 6 e de outro compreendido entre 3,1 e 3,2.

Como faço para encontrar números compreendidos entre dois números inteiros, e dois números decimais, no caso 3,1 e 3,2.

Abraço.
LuizCarlos
Colaborador Voluntário
Colaborador Voluntário
 
Mensagens: 254
Registrado em: Ter Jun 21, 2011 20:39
Formação Escolar: ENSINO MÉDIO
Área/Curso: 1º ano do segundo grau
Andamento: cursando

Re: Questão, número irracional.

Mensagempor MarceloFantini » Sex Mar 16, 2012 22:27

Primeiro, é bom perceber que não existe representação decimal finita de um número irracional, apenas as aproximações por racionais que tem, uma vez que pela definição sabemos que um número racional é a divisão por dois números inteiros. Para esclarecer esse exercício, pense num número cuja raíz quadrada por exemplo esteja entre 5 e 6. Ou seja, 5 < \text{numero} < 6. Isto significa que, elevado ao quadrado, teremos \text{numero}^2 > 25 e \text{numero}^2 < 36 pois respeita a desigualdade. Daí, basta escolher algum que te interesse. Por exemplo, \text{numero}^2 = 30 \implies \text{numero} = \sqrt{30}.

Para facilitar o entendimento, usei "número", mas formalmente escreva alguma letra para denotar álgebra. Perdão pela falta de acento, não tem como no LaTeX.
Futuro MATEMÁTICO
e^{\pi \cdot i} +1 = 0
MarceloFantini
Colaborador Moderador
Colaborador Moderador
 
Mensagens: 3126
Registrado em: Seg Dez 14, 2009 11:41
Formação Escolar: GRADUAÇÃO
Andamento: formado


Voltar para Álgebra Elementar

 



  • Tópicos relacionados
    Respostas
    Exibições
    Última mensagem

Quem está online

Usuários navegando neste fórum: Nenhum usuário registrado e 2 visitantes

 



Assunto: cálculo de limites
Autor: Hansegon - Seg Ago 25, 2008 11:29

Bom dia.

Preciso de ajuda na solução deste problema, pois só chego ao resultado de 0 sobre 0.
Obrigado

\lim_{x\rightarrow-1} x³ +1/x²-1[/tex]


Assunto: cálculo de limites
Autor: Molina - Seg Ago 25, 2008 13:25

\lim_{x\rightarrow-1} \frac{{x}^{3}+1}{{x}^{2}-1}

Realmente se você jogar o -1 na equação dá 0 sobre 0.
Indeterminações deste tipo você pode resolver por L'Hôpital
que utiliza derivada.
Outro modo é transformar o numerador e/ou denominador
para que não continue dando indeterminado.

Dica: dividir o numerador e o denominador por algum valor é uma forma que normalmente dá certo. :y:

Caso ainda não tenha dado uma :idea:, avisa que eu resolvo.

Bom estudo!


Assunto: cálculo de limites
Autor: Guill - Dom Abr 08, 2012 16:03

\lim_{x\rightarrow-1}\frac{x^3+1}{x^2-1}

\lim_{x\rightarrow-1}\frac{(x+1)(x^2-x+1)}{(x+1)(x-1)}

\lim_{x\rightarrow-1}\frac{(x^2-x+1)}{(x-1)}=\frac{-3}{2}